K
Khách

Hãy nhập câu hỏi của bạn vào đây, nếu là tài khoản VIP, bạn sẽ được ưu tiên trả lời.

b) Phương trình hoành độ giao điểm của (D1) và (d2) là:

-x+4=x-4

\(\Leftrightarrow-2x=-8\)

hay x=4

Thay x=4 vào (d1), ta được:

y=-4+4=0

Thay x=0 vào (d1), ta được:

\(y=-0+4=4\)

Thay x=0 vào (d2), ta được:

\(y=0-4=-4\)

Vậy: A(0;4); B(0;-4); C(4;0)

 

a: loading...

b: Tọa độ A là;

y=0 và x+1=0

=>x=-1 và y=0

Tọa độ B là:

y=0 và x*căn 3-3=0

=>x=căn 3 và y=0

Tọa độ C là:

x+1=xcăn 3-3 và y=x+1

=>\(x=\dfrac{-4}{-\sqrt{3}+1}=2+2\sqrt{3}\) và y=3+3căn 3

A(-1;0); B(căn 3;0); \(C\left(2+2\sqrt{3};3+3\sqrt{3}\right)\)

\(AC=\sqrt{\left(2+2\sqrt{3}+1\right)^2+\left(3\sqrt{3}\right)^2}\simeq8,29\)

\(AB=\sqrt{\left(\sqrt{3}+1\right)^2}\simeq2,73\)

\(BC=\sqrt{\left(2+2\sqrt{3}-\sqrt{3}\right)^2+\left(3+3\sqrt{3}\right)^2}\simeq9,0\left(cm\right)\)

\(cosA=\dfrac{AB^2+AC^2-BC^2}{2\cdot AB\cdot AC}\simeq-\dfrac{245}{2487}\)

=>góc A=96 độ

\(cosB=\dfrac{BA^2+BC^2-AC^2}{2\cdot BA\cdot BC}=\dfrac{271}{675}\)

=>góc B=67 độ

=>góc C=17 độ

16 tháng 11 2023

a: loading...

 

b: Tọa độ A là:

\(\left\{{}\begin{matrix}y=0\\3x-1=0\end{matrix}\right.\)

=>\(\left\{{}\begin{matrix}x=\dfrac{1}{3}\\y=0\end{matrix}\right.\)

Vậy: A(1/3;0)

Tọa độ B là:

\(\left\{{}\begin{matrix}y=0\\-x+3=0\end{matrix}\right.\)

=>\(\left\{{}\begin{matrix}y=0\\-x=-3\end{matrix}\right.\Leftrightarrow\left\{{}\begin{matrix}y=0\\x=3\end{matrix}\right.\)

Vậy: B(3;0)

Tọa độ C là:

\(\left\{{}\begin{matrix}3x-1=-x+3\\y=3x-1\end{matrix}\right.\)

=>\(\left\{{}\begin{matrix}4x=4\\y=3x-1\end{matrix}\right.\Leftrightarrow\left\{{}\begin{matrix}x=1\\y=3\cdot1-1=2\end{matrix}\right.\)

Vậy: C(1;2)

c: Gọi \(\alpha\) là góc tạo bởi (d1) với trục Ox

\(tan\alpha=a=3\)

=>\(\alpha\simeq71^033'\)

11 tháng 3 2017

a, HS Tự làm

b, Tìm được C(–2; –3) là tọa độ giao điểm của  d 1  và  d 2

c, Kẻ OH ⊥ AB (CHOx)

S A B C = 1 2 C H . A B = 9 4 (đvdt)

10 tháng 9 2021

\(b,\) Tọa độ giao điểm 2 đường thẳng là:

\(\left\{{}\begin{matrix}y=-2x+4\\y=x+1\end{matrix}\right.\Leftrightarrow\left\{{}\begin{matrix}x+1=-2x+4\\y=x+1\end{matrix}\right.\\ \Leftrightarrow\left\{{}\begin{matrix}x=1\\y=2\end{matrix}\right.\Leftrightarrow A\left(1;2\right)\)

Tọa độ giao điểm 2 đường thẳng với trục hoành là 

\(\left\{{}\begin{matrix}y=0\\\left[{}\begin{matrix}y=-2x+4\\y=x+1\end{matrix}\right.\end{matrix}\right.\Leftrightarrow\left\{{}\begin{matrix}y=0\\\left[{}\begin{matrix}4-2x=0\\x+1=0\end{matrix}\right.\end{matrix}\right.\Leftrightarrow\left\{{}\begin{matrix}y=0\\\left[{}\begin{matrix}x=2\\x=-1\end{matrix}\right.\end{matrix}\right.\)

\(\Leftrightarrow B\left(2;0\right),C\left(-1;0\right)\)

25 tháng 11 2023

a: loading...

b: Phương trình hoành độ giao điểm là:

4x-2=-x+3

=>4x+x=3+2

=>5x=5

=>x=1

Thay x=1 vào y=-x+3, ta được:

\(y=-1+3=2\)

Vậy: M(1;2)

c: Gọi \(\alpha;\beta\) lần lượt là góc tạo bởi (d1),(d2) với trục Ox

(d1): y=4x-2

=>\(tan\alpha=4\)

=>\(\alpha=76^0\)

(d2): y=-x+3

=>\(tan\beta=-1\)

=>\(\beta=135^0\)

d: Thay y=6 vào (d1), ta được:

4x-2=6

=>4x=8

=>x=2

=>A(2;6)

Thay x=6/2=3 vào (d2), ta được:

\(y=-3+3=0\)

vậy: B(3;0)

Vì (d):y=ax+b đi qua A(2;6) và B(3;0) nên ta có hệ phương trình:

\(\left\{{}\begin{matrix}2a+b=6\\3a+b=0\end{matrix}\right.\)

=>\(\left\{{}\begin{matrix}2a+b-3a-b=6-0\\3a+b=0\end{matrix}\right.\Leftrightarrow\left\{{}\begin{matrix}-a=6\\b=-3a\end{matrix}\right.\)

=>\(\left\{{}\begin{matrix}a=-6\\b=-3\cdot\left(-6\right)=18\end{matrix}\right.\)

Vậy: (d): y=-6x+18

e: A(2;6); B(3;0); M(1;2)

\(AM=\sqrt{\left(1-2\right)^2+\left(2-6\right)^2}=\sqrt{17}\)

\(BM=\sqrt{\left(1-3\right)^2+\left(2-0\right)^2}=2\sqrt{2}\)

\(AB=\sqrt{\left(3-2\right)^2+\left(0-6\right)^2}=\sqrt{37}\)

Chu vi tam giác AMB là:

\(C_{AMB}=\sqrt{17}+2\sqrt{2}+\sqrt{37}\)

Xét ΔAMB có 

\(cosAMB=\dfrac{MA^2+MB^2-AB^2}{2\cdot MA\cdot MB}=\dfrac{17+8-37}{2\cdot2\sqrt{2}\cdot\sqrt{17}}=\dfrac{-3}{\sqrt{34}}\)

=>\(\widehat{AMB}\simeq121^0\) và \(sinAMB=\sqrt{1-\left(-\dfrac{3}{\sqrt{34}}\right)^2}=\dfrac{5}{\sqrt{34}}\)

Xét ΔAMB có

\(\dfrac{AB}{sinAMB}=\dfrac{AM}{sinABM}=\dfrac{BM}{sinBAM}\)

=>\(\dfrac{\sqrt{17}}{sinABM}=\dfrac{2\sqrt{2}}{sinBAM}=\sqrt{37}:\dfrac{5}{\sqrt{34}}\)

=>\(sinABM\simeq0,58;\widehat{BAM}\simeq0,4\)

=>\(\widehat{ABM}\simeq35^0;\widehat{BAM}\simeq24^0\)